How do you find the interval of existence for the real function #ln(1+x/(ln(1-x)))#?

3 Answers
Jun 21, 2016

The domain is #(0, 1)#

Explanation:

The domain of the Real valued function of Reals #ln(x)# is #(0, oo)#

So looking at the denominator #ln(1-x)# first, we require:

#1 - x > 0#.

Add #x# to both sides to find #1 > x#, that is #x < 1#

Additionally we require #ln(1-x) != 0# in order that the denominator be non-zero. Hence #x != 0#.

So #x in (-oo, 0) uu (0, 1)#

In order that #1+x/(ln(1-x)) > 0#, we require:

#x/(ln(1-x)) > -1#

Split into cases:

Case #x in (0, 1)#

Then #1-x in (0, 1)# and #ln(1-x) in (-oo, 0)#

Multiply both sides of the inequality by #ln(1-x)# and reverse the inequality (since #ln(1-x) < 0#) to get:

#x < -ln(1-x)#

This inequality holds in the whole interval #(0, 1)#. I may prove later, but here's a plot for now:

graph{(y+ln(1-x))(y-x)=0 [-1.895, 3.105, -0.68, 1.818]}

Case #x in (-oo, 0)#

In this case:

#1 - x > 1#

So:

#ln(1-x) > 0#

Multiply both sides of the required inequality by #ln(1-x)# to get:

#x > -ln(1-x)#

This is false for all #x < 0# so there is no #x < 0# in the domain.

Proof

What is the slope of #-ln(1-x)#?

#d/(dx) -ln(1-x) = 1/(1-x)#

So when #x in (0, 1)# we find:

#1/(1-x) > 1#

and when #x in (-oo, 0)# we find:

#1/(1-x) < 1#

When #x = 0#, #-ln(1-x) = -ln(1) = 0#

So #-ln(1-x)# is steeper than #y=x# when #x > 0# and less steep when #x < 0#. They both pass through #(0, 0)#.

Jun 21, 2016

#log_e(1+x/(log_e(1-x)))#

is feasible only for #0 < x < 1#

Explanation:

#log_e(1+x/(log_e(1-x)))=log_e((log_e(1-x)+x)/(log_e(1-x)))#

The feasibility condition is

#(log_e(1-x)+x)/(log_e(1-x)) > 0#

# 1)# For #0 < x < 1# we have the condition

#log_e(1-x)+x < 0->log_e (1/(e^x(1-x))) > 0#

or #1/(e^x(1-x)) > 1->e^{-x} > 1-x#

but #y=1-x# is tangent to #y=e^{-x}# at #x = 0# and truly

#1-x <= e^{-x} forall x in RR#

#2)#Considering now the condition #1 < x < oo#, the feasibility condition is

# log_e(1-x)+x > 0->log_e(e^x(1-x)) > 0#

or

#1-x>e^{-x}#

but #y=1-x# is tangent to #y=e^{-x}# at #x = 0# and truly

#1-x <= e^{-x} forall x in RR#

#3)# For # -oo > x > 0# we have

#1+x/log_e(1+x) < 0# because #delta/log_e(1+delta) >=1#

Concluding,

#log_e(1+x/(log_e(1-x)))#

is feasible only for #0 < x < 1#

Jun 22, 2016

#( 0, 1 )#.

Explanation:

#ln(1-x)# exists, for x in [-1, 1)#.

#ln(1+x/(ln(1-x)))# exists, for #x/(ln(1-x)) in (-1, 1]#.

Now, as #x to 0#, the given function #to -oo#.

The proof is a tribute to L'Hospital rule..

For #x < 0, ln(1-x)>0 and x/ln(1-x)<-1#.

So, the argument for the given ln function becomes negative. The

function does not exist.

Thus, the answer is (0, 1).. .